The mass of a radioactive substance follows a continuous exponential decay model, with a decay rate parameter of 6.2% per day. Find the half-life of this substance (that is, the time it takes for one-half the original amount in a given sample of this substance to decay). Round your answer to the nearest hundredth.

Answers

Answer 1

Solution

for this case we have the following equation:

[tex]A=A_oe^{kt}_{}[/tex]

the constant would be:

k= -0.062

Then we can do this:

[tex]\frac{1}{2}A_o=A_oe^{-0.062t}[/tex]

solving for t we have:

[tex]\ln (\frac{1}{2})=-0.062t[/tex][tex]t=-\frac{\ln (0.5)}{-0.062}=11.179\text{days}[/tex]

Rounded to the nearest hundredth would be:

11.18 days


Related Questions

(a) Does f (x) have a horizontal asymptote? If so, what is it?(b) Does f (x) have any vertical asymptotes? If so, what are they?

Answers

a) Horizontal asymptotes are horizontal lines that the graph of a function approaches but never touches. To find the horizontal asymptote, we would apply one of the rules which states that

If the degree of the of the denominator is bigger than the degree of the numerator, the horizontal asymptote is the x axis of the graph. It occurs at y = 0

The degree is the largest exponent in the function. Looking at the given function, the degree of the numerator is 2 while the degree of the denominator is 3. Thus,

there is a horizontal asymptote at y = 0

b) The vertical asymptotes are vertical lines which correspond to the zeros of the denominator of rational functions. It is equal to the values of x that make the denominator to be zero. Looking at the given function, (x + 1) cancels out in the numerator and denominator. We are left with (x - 4) and (x + 5). We would equate both terms to zero and solve for x. These values of x would make the denominator to be equal to zero. We have

x - 4 = 0

x = 4

x + 5 = 0

x = - 5

Thus,

there are vertical asymptotes at x = - 5 and x = 4

The price of a train ticket consists of an initial fee of $5 plus a fee of $2.75 per stop. Julia has $21 and would like to travel 50 kilometers. She wants to know the largest number of stops she can afford to buy on a ticketLet S represent the number of stops that Julia buys.1) Which inequality describes this scenario?A. 5+2.75•S ≤ 21 B. 5+2.75•S ≥ 21 C. 5+2.75•S ≤ 50 D. 5+2.75•S ≥ 502) What is the largest number of stops that Julia can afford?

Answers

Let's begin by listing out the information given to us:

Initial fee = $5

Fee per stop = $2.75

Amount with Julia = $21

What is the highest number of stops she can make?

S = the number of stops Julia bought

Julia pays the initial fee of $5. We subtract this from the $21, we have

$ (21 - 5) = $16

Julia has $16 left to buy her stops. She cannot spend beyond the amount of money with her (altogether $21). She spends lesser than or equal to $21 (≤ $21)

The inequality that describes this scenario is given by:

initial fee + fee per stop * number of stops ≤ 21

5 + 2.75 * S ≤ 21

Hence, option A is the correct answer

What is the largest number of stops that Julia can afford?

This is gotten by dividing the amount left after subtracting the initial fee by the fee per stop

n = 16/2.75 = 5.82 = 5 stops (rounding downwards)

We round downwards because the number of stops must be a whole number and it must be lesser than or equal to $21 altogether

2) Given that XY || AC, what is YC if BX = 10, BA = 15, and BY = 8?A) 4 B) 6 C)8D)12

Answers

We can see that triangles ABC and AXY are congruent

This means that

[tex]\frac{AX}{BX}=\frac{YC}{BY}[/tex]

Now, we know that BX=10, BY=8 and BA=AX+BX, hence AX=BA-BX, we have

[tex]\frac{BA-BX}{10}=\frac{YC}{8}[/tex]

now, since BA-BX=15-10, BA-BX=5, it yields,

[tex]\frac{5}{10}=\frac{YC}{8}[/tex]

Now, we need to isolate YC, this is given by

[tex]YC=8(\frac{5}{10})[/tex]

Since

[tex]\frac{5}{10}=\frac{5\cdot1}{5\cdot2}=\frac{1}{2}[/tex]

we have that

[tex]\begin{gathered} YC=8(\frac{1}{2}) \\ YC=\frac{8}{2} \\ YC=4 \end{gathered}[/tex]

hence, the answer is YC=4, which corresponds to A).

polinomials (x + 3)2

Answers

Given the following question:

[tex](x+3)2[/tex]

(x + 3)2

First, we flip the polynomial:

(x + 3)2 = 2(x + 3)

2(x + 3)

Next, we apply the distributive law where we multiply 2 by x and 3.

2 × x = 2x

2 × 3 = 6

2x + 6

Expression cannot be simplified any further:

= 2x + 6

According to the graph, what is the value of the constant in the equation below? 2- 18+ Height = Constant Wiat 16+ (0.5,1.6) 12+ Height (0.8.1) 0.8 + (1.6.05) (2.0.4) 12 14 16 18 2

Answers

Liyah, this is the solution:

• Height = Constant/Width

,

• Height * Width = Constant (you need to multiply each ordered pair )

Therefore,

Constant = 1.6 * 0.5

Constant = 0.8

Constant = 0.4 * 2

Constant = 0.8

The correct answer is C. 0.8

The number of adults living in homes on a randomly selected city block is described by the following probability distribution. Number of adults, x1 ,2,3,4 or moreProbability, P(x) 0.250.500.15??? What is the probability that 4 or more adults reside at a randomly selected home?(A) 0.10(B) 0.15 (C) 0.25(D) 0.50 (E) 0.90

Answers

The answer is letter A. 0.1

because the probability = 1

So Probability of select 4 adults = 1 - 0.25 - 0.5 - 0.15

= 0.1

find the smallest non negative value for x in degrees that makes the equation cot (x) = √3 true.

Answers

Given:

cot (x) = √3

To find the smallest non-negative value for x in degrees:

So, we get

[tex]\begin{gathered} \cot x=\sqrt{3} \\ \cot x=\cot (30^{\circ}) \\ x=30^{\circ} \end{gathered}[/tex]

Hence, the answer is,

[tex]x=30^{\circ}[/tex]

Y 3+ 2+ 1+ -4 -3 -2 -1 1 2 3 -1- -2 -3+ -4 47 What is the slope of the line?

Answers

To find the slope of the line, we will follow the steps given below:

Step 1: select two points on the graph

(0, -1) and (4,2)

Step 2: Apply the slope formula:

[tex]\text{slope}=\frac{y_2-y_1}{x_2-x_1}[/tex]

=>

[tex]\text{slope}=\frac{2-(-1)}{4-0}=\frac{2+1}{4}=\frac{3}{4}[/tex]

The slope of the graph is:

[tex]\frac{3}{4}[/tex]

What is an equation of the line parallel to the line on the graph that passes through (2,25)?

Answers

y=4x+17

Explanation

Step 1

2 equations of lines are parallel if the slope is the same, so

a) find the slope of the graphed line

the slope of a line can by calculated by using

[tex]\begin{gathered} slope=\frac{change\text{ in y }}{change\text{ in x}}=\frac{y_2-y_1}{x_2-x_1} \\ where \\ P1(x_1,y_1) \\ and \\ P2(x_2,y_2) \\ are\text{ 2 points from the line} \end{gathered}[/tex]

so

pick up 2 points from the the line and let

[tex]\begin{gathered} P1(0,10) \\ P2(10,50) \end{gathered}[/tex]

replace and evaluate

[tex]\begin{gathered} slope=\frac{change\text{\imaginaryI ny}}{change\text{\imaginaryI nx}}=\frac{y_{2}-y_{1}}{x_{2}-x_{1}} \\ slope=\frac{50-10}{10-0}=\frac{40}{10}=4 \end{gathered}[/tex]

hence, the slope of the line is 4

Step 2

now, using the slope and a point we can find the equation of the line

use the point-slope formula, it says

[tex]\begin{gathered} y-y_1=m(x-x_1) \\ where\text{ m is the slope} \\ (x_1,y_1)\text{ is a point from the line} \end{gathered}[/tex]

so

a)let

[tex]\begin{gathered} P1(2,25) \\ sloipe=4 \end{gathered}[/tex]

b) now ,replace and solve for y

[tex]\begin{gathered} y-y_{1}=m(x-x_{1}) \\ y-25=4(x-2) \\ y-25=4x-8 \\ add\text{ 25 in both sides} \\ y-25+25=4x-8+25 \\ y=4x+17 \end{gathered}[/tex]

so, the answer is

y=4x+17

I

the radius of a circle is 4 centimeters. what is the diameter? give the exact answer in simplest form

Answers

we have that

the diameter is two times the radius

so

in this problem

D=2r

D=2(4)=8 cm

diameter is 8 cm

Which number is not equal to 225%?its is exercise number 5

Answers

To do this, you can first convert the percentage form to its decimal form, like this

[tex]225\text{\%}=\frac{225}{100}=2.25[/tex]

Now, you can convert the numbers that are possible answers into their decimal form, like this

Option A.

[tex]2\frac{1}{4}=\frac{2\cdot8+1}{4}=\frac{8+1}{4}=\frac{9}{4}=2.25[/tex]

Option B.

[tex]\frac{9}{4}=2.25[/tex]

Option C.

[tex]\frac{50}{40}=\frac{5\cdot10}{4\cdot10}=\frac{5}{4}=1.25[/tex]

Option D.

[tex]\frac{45}{20}=\frac{9\cdot5}{4\cdot5}=\frac{9}{4}=2.25[/tex]

Therefore, the number that is not equal to 225% is 50/40 and the correct answer is C. 50/40.

9. At last Friday's soccer game there were a total of 673 fans in attendance, including students and non-students.Let x represent the number of students, and y represent non-students. Which of the following statements couldrepresent the number of fans in attendance. Select all that apply.a. x + y = 673b. 335 and 138c. 335 and 338d. x=y - 673e. y = -x + 673f. 273 and 400

Answers

The answer is A

From the question:

Total fans in attendence = 673

x = number of students

y= non - students

Total of fans in attendance =

x + y= 673

5Jamal's band learns lots of new songs.The band learns a new song every fourdays. At this rate, how many new songswill the band learn in four weeks?LAsongs

Answers

Given that The band learns a new song every four days.

We need to find the number of new songs for four weeks.

We know that one week= 7 days

[tex]1\text{ w}eek\text{ = 7 days }[/tex]

Multiply by 4 to find the number of days in four weeks.

[tex]1\times4\text{ w}eek\text{ = 7 }\times4\text{ days }[/tex]

[tex]4\text{ w}eeks\text{= 28days }[/tex]

We need to find the number of new songs that the band learns in 28 days.

Divide 28 by 4 to find the number of songs since the band learns one new song every 4 days.

[tex]\frac{28}{4}=7\text{ songs}[/tex]

The band learns 7 songs in four weeks.

help me asap please on this math question

Answers

Equations showing direct variations are 2x = y and y = 1.8c

Direct Variation exists between two variables when one variable is directly dependent to another variable means change in one variable will create change in other one also and vice versa.

Two variable increase or decrease by the same factor.

Suppose x and y is that are in direct variation then you can write

y ∝ x

where, "∝" denotes proportionality

removing proportionality sign by constant then you can write

y = k x , where k is constant and can hold any real value

From the following equation ,

2x = y with 2 as constant and

y = 1.8x with 1.8 as constant shows direct variations

To know more about Direct variation here

https://brainly.com/question/18087321

#SPJ1

Consider the following simple statements: p: Your shirt is tucked into your pants. q: Your pants are tucked into your shirt.What is the symbolic form of the statement: "If your shirt isn't tucked into your pants then your pants are tucked into your shirt."Select the correct answer below:∼q⟹p∼q⟹∼pp⟹∼q∼p⟹q

Answers

SOLUTION

We are asked the symbolic form of "If your shirt isn't tucked into your pants then your pants are tucked into your shirt."

This simply means the negation of p implies q.

p implies q is represented as p⟹q

Then the negation of p implies q will be ∼p⟹q.

Therefore, the correct answer is ∼p⟹q

8) Suppose y varies inversely as x, if y = 7 when x = 6then find y when x = -21.

Answers

Suppose y varies inversely as x, if y = 7 when x = 6

then find y when x = -21.

we know that

A relationship between two variables, x, and y, represent an inverse variation if it can be expressed in the form

y*x=k

where

k is the constant of proportionality

step 1

Find the value of k

we have

y=7, x=6

k=7*6

k=42

the equation is

y*x=42

step 2

For x=-21

substitute

y*(-21)=42

y=42/(-21)

y=-2

12x+18 rewrite using distributive property

Answers

we have

12x+18

REmember that

12=(2^2)*(3)

18=(2)*(3^2)

substitute

(2^2)*(3)x+(2)*(3^2)

Factor (2)*(3)=6

6(2x+3)

therefore

the answer is

6(2x+3)

Suppose that the functions fand g are defined for all real numbers x as follows.f(x)=x+5g(x)=2x²Write the expressions for (g+f)(x) and (g–f)(x) and evaluate (g.f)(-3).

Answers

The expression (g+f)(x) is equal to g(x)+ f(x), (g-f)(x) is equal to g(x) f(x) and the expression (g*f)(-3) is equal to g(-3)*f(-3).

Then, we have

[tex](g+f)(x)=g(x)+f(x)=2x^2+x+5[/tex]

Similarly,

[tex](g-f)(x)=g(x)f(x)=2x^2-(x+5)=2x^2-x-5[/tex]

And finally,

[tex]\begin{gathered} (g\cdot f)(-3)=g(-3)\cdot f(-3)=2(-3)^2\cdot(-3+5) \\ (g\cdot f)(-3)=2(9)\cdot(2) \\ (g\cdot f)(-3)=36 \end{gathered}[/tex]

In summary, the answers are:

[tex]\begin{gathered} (g+f)(x)=2x^2+x+5 \\ (g-f)(x)=2x^2-x-5 \\ (g\cdot f)(-3)=36 \end{gathered}[/tex]

3 2 1 -3-2- 1 2 3 2 -3 Domain: (-3,3] Range: [-2, 2] Domain: (-2, 2] Range: [-3,3] Domain: (-2,-3) Range: (2,3) Domain: {-2, -1, 0, 1, 2} Range: {-3, -2, - 1, 0, 1, 2, 3} None of the above NON

Answers

The domain is [ -2, 2]

and the range is [-3, 3]

Use the following function for questions # 1 - # 5:f(x) =x?- 14x + 44#1: Find the X value of the turning point.

Answers

The given function is

f(x) = x^2 - 14x + 44

To find the turning point, we would differentiate the function, equate the derivative to zero and solve for x. We have

f'(x) = 2x - 14

Equating it to zero, we have

2x - 14 = 0

2x = 14

x = 14/2

x = 7

The value of x of the turnng point is 7

An empty swimming pool needs to be filled to the top. The pool is shaped like a cylinder with a diameter of 9 m and a depth of 1.1 m. Suppose water is pumped into the pool at a rate of 13 m^3 per hour. How many hours will it take to fill the empty pool?

Use the value 3.14 for pi, and round your answer to the nearest hour. Do not round any intermediate computations.

Answers

Answer:

πr2h

volume of cylinder

3.14×3×3×1.1=31.086m^3

1hour=13m^3

31.086m^3

divide 31.086÷13=2.3912hours

A __ is a polynomial with one term.

Answers

Answer:

Monomial

Step-by-step explanation:

A polynomial that consists of exactly one term is called monomial.

Examples are 3, 10x², xy,...

So the answer is: Monomial

What is 132% as a decimal?

Answers

Step 1: Problem

What is 132% as a decimal?​

.132 is your answer because if you where getting the percent off of something you would do that number X divided by .132 to get the answer.

What is the measure of

Answers

In the parallelogram ABCD,

Angle D is 145 degree.

In the parallelogram adjacent angles sum is 180 degree.

In the given parallelogram ABCD , angle D and angle C are adjacent.

[tex]\begin{gathered} \angle D+\angle C=180^{\circ} \\ 145^{\circ}+\angle C=180^{\circ} \\ \angle C=180^{\circ}-145^{\circ} \\ \angle C=35^{\circ} \end{gathered}[/tex]

Answer: Option B) 35 degree.

Identify the domain and range for the given relation. Indicate whether the relation is a function or not andexplain

Answers

Given :

Domain is:

[tex]D\colon\mleft\lbrace0,-1,1\mright\rbrace[/tex]

Range is:

[tex]R\colon\mleft\lbrace0,1,2\mright\rbrace[/tex]

Here is one output for one input.

hi I need help ;]]] ‍❄️‍❄️‍❄️

Answers

The order from least to greatest is -4.7,-4,-31/8, [tex]-3\frac{1}{8}[/tex]

What is Fraction?

A fraction represents a part of a whole.

The given integers are -31/8,-4.7, -4 and -3 1/8

Now let us simplify the fraction values

-31/8=-3.875

-4.7

-4 and

     [tex]-3\frac{1}{8}[/tex]=-24+1/8=-23/8=-2.875

As there is a negative sign, the smallest number with negative sign will be greatest and largest number with negative sign is smaller.

So -4.7,-4, -3.875, -2.875

-4.7,-4,-31/8, [tex]-3\frac{1}{8}[/tex]

Hence the order from least to greatest is -4.7,-4,-31/8, [tex]-3\frac{1}{8}[/tex]

To learn more on Fractions click:

https://brainly.com/question/10354322

#SPJ1

Simplify the expression cos x/ cot x.a. cos xb. tan xc. sin xd. cos²x/sin x

Answers

cosx/cotx = cosx *tan x =cosx (sinx/cosx) = sin x

Answer

c. sin x

You flip a coin and roll a die. The table shows the sample space.12 3 4 5 6Heads(H) H-1 H-2 H-3 H-4H-5H-6Tails(T) T-1 T-2 T-3 T-4 T-5 T-6What is the probability of getting a head or a tail and anven number?Answer as a reduced fraction in the form ab.

Answers

You flip a coin and roll a die. The table shows the sample space.

1

2 3 4 5 6

Heads(H) H-1 H-2 H-3 H-4H-5H-6

Tails(T) T-1 T-2 T-3 T-4 T-5 T-6

What is the probability of getting a head or a tail and an

even number?

we know that

The probability of an event is the ratio of the size of the event space to the size of the sample space.

The size of the sample space is the total number of possible outcomes

The event space is the number of outcomes in the event you are interested in.

In this problem

the size of the sample space is (6+6+6)=18

the size of the event space is equal to (6+6+3)=15

REmember that an even number are (2,4 and 6)

so

the probability is equal to

P=15/18

simplify

P=5/6

therefore

the answer is5/6

help me asap please!!! no explanation just the process and answer

Answers

To find out the determinant, multiply in cross

so

(3)*(-2)-(5)*(-7)=-6+35=29

therefore

the answer is 29

What is the surface area of the regular pyramid below?A. 648 sq. unitsB. 552 sq. unitsC. 396 sq. unitsD. 522 sq. units

Answers

Step 1:

Concept: Calculate the area of each face and add all together to get the surface area of the pyramid.

The regular pyramid below have 4 triangles and a square

Step 2: Apply the area formula to find the area of the 4 triangles and a square.

[tex]\begin{gathered} \text{Area of a triangle = }\frac{Base\text{ x Height}}{2} \\ \text{Area of the square base = Length x Length} \end{gathered}[/tex]

Step 3:

Given data for the triangle

Height = 21

Base = 12

[tex]\begin{gathered} Area\text{ of a triangle = }\frac{Base\text{ x Height}}{2} \\ =\text{ }\frac{21\text{ x 12}}{2} \\ =\text{ }\frac{252}{2} \\ =126\text{ sq. units} \\ \text{Area of the four triangles = 4 x 126 = 504 sq. units} \end{gathered}[/tex]

Step 4: Find the area of the square

Given data for the square

Length = 12

Area = length x length = 12 x 12 = 144 sq. units

Step 5: Add the area of the four triangles and the square.

Surface area of the regular pyramid = 504 + 144

= 648 sq. units

Other Questions
what's the solution to this system The repetition of beginning sounds in words is Find the value of r so that the line through (-4, r) and (-8, 3) has a slope of -5. According to the graph of H(w) below, what happens when w gets very large?H)5.6.20.00)A. H(w) gets very large.B. H(w) approaches a vertical asymptote.C. H(w) equals zero.D. H(w) gets very smallSUBMIT 100 POINTS!! I NEED THIS KNOWW!!!!The number line shows the distance in meters of two birds, A and B, from a worm located at point X:A horizontal number line extends from negative 3 to positive 3. The point labeled as A is at negative 2.5, the point 0 is labeled as X, and the point labeled B is at 2.5.Write an expression using subtraction to find the distance between the two birds.Show your work and solve for the distance using additive inverses. Consider the function g. 9(-) = 6() For the x-values given in the table below, determine the corresponding values of g(x) and plot each point on the graph.. -1 0 1 2 g(x) Drawing Tools Click on a tool to begin drawing * Delete Undo Reset Select Point 14 13 12 11 10 9 00 reserved. Select the point that satisfies y x-3x+2. is this relation a function? Justify your answer Find the circumference of the circle. Give the exact circumference and then an approximation. Use i 3.14. diamater of 17cm Admission to the fair costs $6.00. Each ride costs you$0.50. You have $22.00 to spend at the fair on rides and admission. Express the number of tickets you can buy as an inequality. lc. A student makes a claim that states the first hill of a roller coaster is always the tallest(Point A) provided there is no mechanical assistance at any point after Point A on the track.Do you accept or reject her claim? Explain your answer. (2 points) Following the French defeat, Native Americans were happy to trade with the British As the business cycle descends, it depicts the state of the economy. In this phase, it is said to beA branching outB crestingC contractingD expanding The boxplot below shows salaries for Construction workers and Teachers.ConstructionTeacher2025465030 35 40Salan (thousands of S)If a person is making the median salary for a construction worker, they are making more than what percentage ofTeachers?They are making more than% of Teachers.Check Answer What is the role of an enzyme in the body Translate the following into algebraic equation and solve: Twice the sum of a number and five is equal to 40. For the rotation -1046, find the coterminal angle from 0 < O < 360, the quadrant and the reference angle. Need answer to pictured problem! The answer should be in reference to trig identities The library is installing newShelves. If the carpenter has4/5 yard of wood to makeinto 2 shelves of equal length,how long will each shelf be?Menth A tractor travels at a constant speed of 6m/s. Find the power supplied by the engine if it can supply a maximum force of 467 kN.